Difference between revisions of "1990 AHSME Problems/Problem 2"

(Created page with "== Problem == <math>\left(\frac{1}{4}\right)^{-\tfrac{1}{4}}=</math> <math>\text{(A) } -16\quad \text{(B) } -\sqrt{2}\quad \text{(C) } -\frac{1}{16}\quad \text{(D) } \frac{1}{2...")
 
(Solution)
 
Line 10: Line 10:
  
 
== Solution ==
 
== Solution ==
<math>\fbox{E}</math>
+
<math>\left(\frac14\right)^{-\tfrac14}=4^{\tfrac14}=\sqrt[4]4=\sqrt{\sqrt4}=\sqrt2</math> which is <math>\fbox{E}</math>
  
 
== See also ==
 
== See also ==

Latest revision as of 03:38, 4 February 2016

Problem

$\left(\frac{1}{4}\right)^{-\tfrac{1}{4}}=$

$\text{(A) } -16\quad \text{(B) } -\sqrt{2}\quad \text{(C) } -\frac{1}{16}\quad \text{(D) } \frac{1}{256}\quad \text{(E) } \sqrt{2}$

Solution

$\left(\frac14\right)^{-\tfrac14}=4^{\tfrac14}=\sqrt[4]4=\sqrt{\sqrt4}=\sqrt2$ which is $\fbox{E}$

See also

1990 AHSME (ProblemsAnswer KeyResources)
Preceded by
Problem 1
Followed by
Problem 3
1 2 3 4 5 6 7 8 9 10 11 12 13 14 15 16 17 18 19 20 21 22 23 24 25 26 27 28 29 30
All AHSME Problems and Solutions

The problems on this page are copyrighted by the Mathematical Association of America's American Mathematics Competitions. AMC logo.png